Show that the sequence $a_1=3$ and $a_{n+1}=frac{3(1+a_n)}{3+a_n}$ converges to $sqrt3$











up vote
1
down vote

favorite












First of all let´s see how it goes: $3,frac{3(1+3)}{3+3}=frac{12}{6}=2,frac{3(1+2)}{3+2}=frac{9}{5}=1.8,...$ We see that it's decreasing.



What i need to show first is that the sequence decreases. To do that, i´d say:



$$a_n-a_{n+1}>0Rightarrow a_n-{frac{3(1+a_n)}{3+a_n}}>0Rightarrow a_n^2>3Rightarrow a_n>sqrt{3}$$ But i think this is like say that $a_n$ converges to $sqrt3$, which is asked to be proved. So, How can i show this sequence decreases?. And to show it's bounded?










share|cite|improve this question
























  • Still study the difference $a_n -sqrt 3$.
    – xbh
    Nov 16 at 2:15















up vote
1
down vote

favorite












First of all let´s see how it goes: $3,frac{3(1+3)}{3+3}=frac{12}{6}=2,frac{3(1+2)}{3+2}=frac{9}{5}=1.8,...$ We see that it's decreasing.



What i need to show first is that the sequence decreases. To do that, i´d say:



$$a_n-a_{n+1}>0Rightarrow a_n-{frac{3(1+a_n)}{3+a_n}}>0Rightarrow a_n^2>3Rightarrow a_n>sqrt{3}$$ But i think this is like say that $a_n$ converges to $sqrt3$, which is asked to be proved. So, How can i show this sequence decreases?. And to show it's bounded?










share|cite|improve this question
























  • Still study the difference $a_n -sqrt 3$.
    – xbh
    Nov 16 at 2:15













up vote
1
down vote

favorite









up vote
1
down vote

favorite











First of all let´s see how it goes: $3,frac{3(1+3)}{3+3}=frac{12}{6}=2,frac{3(1+2)}{3+2}=frac{9}{5}=1.8,...$ We see that it's decreasing.



What i need to show first is that the sequence decreases. To do that, i´d say:



$$a_n-a_{n+1}>0Rightarrow a_n-{frac{3(1+a_n)}{3+a_n}}>0Rightarrow a_n^2>3Rightarrow a_n>sqrt{3}$$ But i think this is like say that $a_n$ converges to $sqrt3$, which is asked to be proved. So, How can i show this sequence decreases?. And to show it's bounded?










share|cite|improve this question















First of all let´s see how it goes: $3,frac{3(1+3)}{3+3}=frac{12}{6}=2,frac{3(1+2)}{3+2}=frac{9}{5}=1.8,...$ We see that it's decreasing.



What i need to show first is that the sequence decreases. To do that, i´d say:



$$a_n-a_{n+1}>0Rightarrow a_n-{frac{3(1+a_n)}{3+a_n}}>0Rightarrow a_n^2>3Rightarrow a_n>sqrt{3}$$ But i think this is like say that $a_n$ converges to $sqrt3$, which is asked to be proved. So, How can i show this sequence decreases?. And to show it's bounded?







sequences-and-series limits






share|cite|improve this question















share|cite|improve this question













share|cite|improve this question




share|cite|improve this question








edited Nov 16 at 16:42

























asked Nov 16 at 2:10









Tom Arbuckle

366




366












  • Still study the difference $a_n -sqrt 3$.
    – xbh
    Nov 16 at 2:15


















  • Still study the difference $a_n -sqrt 3$.
    – xbh
    Nov 16 at 2:15
















Still study the difference $a_n -sqrt 3$.
– xbh
Nov 16 at 2:15




Still study the difference $a_n -sqrt 3$.
– xbh
Nov 16 at 2:15










4 Answers
4






active

oldest

votes

















up vote
1
down vote













Hint. If you can prove that it has some limit $L$ then you can take the limit of both sides of the recursive definition and conclude ...






share|cite|improve this answer




























    up vote
    0
    down vote













    Hint:




    • Let $f(x)=frac{3(1+x)}{3+x}$. Prove that $f(x) le x$ for $x ge sqrt 3$


    • Prove that $x ge sqrt 3 implies f(x) ge sqrt 3$



    Conclude that $a_n$ is decreasing and bounded below and hence converges. Since $f$ is continuous, $a_n$ converges to fixed point of $f$.






    share|cite|improve this answer






























      up vote
      0
      down vote













      You can use induction to show $a_n>a_{n+1}$:
      $$1) a_1=3>2=a_2;\
      2) text{assume} a_{n-1}color{red}>a_n;\
      3) a_{n}=frac{3(a_{n-1}+1)}{3+a_{n-1}}=3-frac{6}{3+a_{n-1}}color{red}>3-frac6{3+a_n}=frac{3(a_n+1)}{3+a_n}=a_{n+1}.$$






      share|cite|improve this answer




























        up vote
        0
        down vote













        We have $a_1>0$ and from the definition of $a_{n+1}$ we have $a_n>0implies a_{n+1}>0 $ for any $n$. So by induction we have $a_n>0$ for all $nin Bbb N.$



        We have $a_1>sqrt 3$. If $a_n>sqrt 3$ then, since $a_{n+1}>0,$ we have $$a_{n+1}>sqrt 3iff a_{n+1}^2>3iff$$ $$iff frac {3^2(1+a_n)^2}{(3+a_n)^2}>3iff$$ $$iff 3^2(1+a_n)^2 -3(3+a_n)^2>0iff$$ $$iff 6(a_n^2-3)>0.$$ So by induction we have $a_n>sqrt 3$ for all $nin Bbb N.$



        Since $3+a_n>a_n>0$ we have $$a_n>a_{n+1} iff (3+a_n)left(a_n-frac {3(1+a_n)}{3+a_n}right)>0 iff$$ $$iff 3a_n+a_n^2> 3+3a_niff a_n^2>3,$$ and we have already shown that $a_n^2>3.$ So $a_n>a_{n+1}$ for all $n.$



        Since $(a_n)_n$ is a decreasing sequence of positive terms it has a limit $Lgeq 0$ . And we have $$L=lim_{nto infty}a_{n+1}=lim_{nto infty}frac {3(1+a_n)}{3+a_n}= frac {3(1+L)}{3+L}.$$ So $L=frac {3(1+L)}{3+L}, $
        which implies $L^2=3.$ Now $(Lgeq 0land L^2=3)implies L=sqrt 3.$






        share|cite|improve this answer





















          Your Answer





          StackExchange.ifUsing("editor", function () {
          return StackExchange.using("mathjaxEditing", function () {
          StackExchange.MarkdownEditor.creationCallbacks.add(function (editor, postfix) {
          StackExchange.mathjaxEditing.prepareWmdForMathJax(editor, postfix, [["$", "$"], ["\\(","\\)"]]);
          });
          });
          }, "mathjax-editing");

          StackExchange.ready(function() {
          var channelOptions = {
          tags: "".split(" "),
          id: "69"
          };
          initTagRenderer("".split(" "), "".split(" "), channelOptions);

          StackExchange.using("externalEditor", function() {
          // Have to fire editor after snippets, if snippets enabled
          if (StackExchange.settings.snippets.snippetsEnabled) {
          StackExchange.using("snippets", function() {
          createEditor();
          });
          }
          else {
          createEditor();
          }
          });

          function createEditor() {
          StackExchange.prepareEditor({
          heartbeatType: 'answer',
          convertImagesToLinks: true,
          noModals: true,
          showLowRepImageUploadWarning: true,
          reputationToPostImages: 10,
          bindNavPrevention: true,
          postfix: "",
          imageUploader: {
          brandingHtml: "Powered by u003ca class="icon-imgur-white" href="https://imgur.com/"u003eu003c/au003e",
          contentPolicyHtml: "User contributions licensed under u003ca href="https://creativecommons.org/licenses/by-sa/3.0/"u003ecc by-sa 3.0 with attribution requiredu003c/au003e u003ca href="https://stackoverflow.com/legal/content-policy"u003e(content policy)u003c/au003e",
          allowUrls: true
          },
          noCode: true, onDemand: true,
          discardSelector: ".discard-answer"
          ,immediatelyShowMarkdownHelp:true
          });


          }
          });














           

          draft saved


          draft discarded


















          StackExchange.ready(
          function () {
          StackExchange.openid.initPostLogin('.new-post-login', 'https%3a%2f%2fmath.stackexchange.com%2fquestions%2f3000622%2fshow-that-the-sequence-a-1-3-and-a-n1-frac31a-n3a-n-converges-to%23new-answer', 'question_page');
          }
          );

          Post as a guest















          Required, but never shown

























          4 Answers
          4






          active

          oldest

          votes








          4 Answers
          4






          active

          oldest

          votes









          active

          oldest

          votes






          active

          oldest

          votes








          up vote
          1
          down vote













          Hint. If you can prove that it has some limit $L$ then you can take the limit of both sides of the recursive definition and conclude ...






          share|cite|improve this answer

























            up vote
            1
            down vote













            Hint. If you can prove that it has some limit $L$ then you can take the limit of both sides of the recursive definition and conclude ...






            share|cite|improve this answer























              up vote
              1
              down vote










              up vote
              1
              down vote









              Hint. If you can prove that it has some limit $L$ then you can take the limit of both sides of the recursive definition and conclude ...






              share|cite|improve this answer












              Hint. If you can prove that it has some limit $L$ then you can take the limit of both sides of the recursive definition and conclude ...







              share|cite|improve this answer












              share|cite|improve this answer



              share|cite|improve this answer










              answered Nov 16 at 2:15









              Ethan Bolker

              39.1k543102




              39.1k543102






















                  up vote
                  0
                  down vote













                  Hint:




                  • Let $f(x)=frac{3(1+x)}{3+x}$. Prove that $f(x) le x$ for $x ge sqrt 3$


                  • Prove that $x ge sqrt 3 implies f(x) ge sqrt 3$



                  Conclude that $a_n$ is decreasing and bounded below and hence converges. Since $f$ is continuous, $a_n$ converges to fixed point of $f$.






                  share|cite|improve this answer



























                    up vote
                    0
                    down vote













                    Hint:




                    • Let $f(x)=frac{3(1+x)}{3+x}$. Prove that $f(x) le x$ for $x ge sqrt 3$


                    • Prove that $x ge sqrt 3 implies f(x) ge sqrt 3$



                    Conclude that $a_n$ is decreasing and bounded below and hence converges. Since $f$ is continuous, $a_n$ converges to fixed point of $f$.






                    share|cite|improve this answer

























                      up vote
                      0
                      down vote










                      up vote
                      0
                      down vote









                      Hint:




                      • Let $f(x)=frac{3(1+x)}{3+x}$. Prove that $f(x) le x$ for $x ge sqrt 3$


                      • Prove that $x ge sqrt 3 implies f(x) ge sqrt 3$



                      Conclude that $a_n$ is decreasing and bounded below and hence converges. Since $f$ is continuous, $a_n$ converges to fixed point of $f$.






                      share|cite|improve this answer














                      Hint:




                      • Let $f(x)=frac{3(1+x)}{3+x}$. Prove that $f(x) le x$ for $x ge sqrt 3$


                      • Prove that $x ge sqrt 3 implies f(x) ge sqrt 3$



                      Conclude that $a_n$ is decreasing and bounded below and hence converges. Since $f$ is continuous, $a_n$ converges to fixed point of $f$.







                      share|cite|improve this answer














                      share|cite|improve this answer



                      share|cite|improve this answer








                      edited Nov 17 at 1:25

























                      answered Nov 16 at 18:01









                      lhf

                      161k9165384




                      161k9165384






















                          up vote
                          0
                          down vote













                          You can use induction to show $a_n>a_{n+1}$:
                          $$1) a_1=3>2=a_2;\
                          2) text{assume} a_{n-1}color{red}>a_n;\
                          3) a_{n}=frac{3(a_{n-1}+1)}{3+a_{n-1}}=3-frac{6}{3+a_{n-1}}color{red}>3-frac6{3+a_n}=frac{3(a_n+1)}{3+a_n}=a_{n+1}.$$






                          share|cite|improve this answer

























                            up vote
                            0
                            down vote













                            You can use induction to show $a_n>a_{n+1}$:
                            $$1) a_1=3>2=a_2;\
                            2) text{assume} a_{n-1}color{red}>a_n;\
                            3) a_{n}=frac{3(a_{n-1}+1)}{3+a_{n-1}}=3-frac{6}{3+a_{n-1}}color{red}>3-frac6{3+a_n}=frac{3(a_n+1)}{3+a_n}=a_{n+1}.$$






                            share|cite|improve this answer























                              up vote
                              0
                              down vote










                              up vote
                              0
                              down vote









                              You can use induction to show $a_n>a_{n+1}$:
                              $$1) a_1=3>2=a_2;\
                              2) text{assume} a_{n-1}color{red}>a_n;\
                              3) a_{n}=frac{3(a_{n-1}+1)}{3+a_{n-1}}=3-frac{6}{3+a_{n-1}}color{red}>3-frac6{3+a_n}=frac{3(a_n+1)}{3+a_n}=a_{n+1}.$$






                              share|cite|improve this answer












                              You can use induction to show $a_n>a_{n+1}$:
                              $$1) a_1=3>2=a_2;\
                              2) text{assume} a_{n-1}color{red}>a_n;\
                              3) a_{n}=frac{3(a_{n-1}+1)}{3+a_{n-1}}=3-frac{6}{3+a_{n-1}}color{red}>3-frac6{3+a_n}=frac{3(a_n+1)}{3+a_n}=a_{n+1}.$$







                              share|cite|improve this answer












                              share|cite|improve this answer



                              share|cite|improve this answer










                              answered Nov 17 at 3:33









                              farruhota

                              17.7k2736




                              17.7k2736






















                                  up vote
                                  0
                                  down vote













                                  We have $a_1>0$ and from the definition of $a_{n+1}$ we have $a_n>0implies a_{n+1}>0 $ for any $n$. So by induction we have $a_n>0$ for all $nin Bbb N.$



                                  We have $a_1>sqrt 3$. If $a_n>sqrt 3$ then, since $a_{n+1}>0,$ we have $$a_{n+1}>sqrt 3iff a_{n+1}^2>3iff$$ $$iff frac {3^2(1+a_n)^2}{(3+a_n)^2}>3iff$$ $$iff 3^2(1+a_n)^2 -3(3+a_n)^2>0iff$$ $$iff 6(a_n^2-3)>0.$$ So by induction we have $a_n>sqrt 3$ for all $nin Bbb N.$



                                  Since $3+a_n>a_n>0$ we have $$a_n>a_{n+1} iff (3+a_n)left(a_n-frac {3(1+a_n)}{3+a_n}right)>0 iff$$ $$iff 3a_n+a_n^2> 3+3a_niff a_n^2>3,$$ and we have already shown that $a_n^2>3.$ So $a_n>a_{n+1}$ for all $n.$



                                  Since $(a_n)_n$ is a decreasing sequence of positive terms it has a limit $Lgeq 0$ . And we have $$L=lim_{nto infty}a_{n+1}=lim_{nto infty}frac {3(1+a_n)}{3+a_n}= frac {3(1+L)}{3+L}.$$ So $L=frac {3(1+L)}{3+L}, $
                                  which implies $L^2=3.$ Now $(Lgeq 0land L^2=3)implies L=sqrt 3.$






                                  share|cite|improve this answer

























                                    up vote
                                    0
                                    down vote













                                    We have $a_1>0$ and from the definition of $a_{n+1}$ we have $a_n>0implies a_{n+1}>0 $ for any $n$. So by induction we have $a_n>0$ for all $nin Bbb N.$



                                    We have $a_1>sqrt 3$. If $a_n>sqrt 3$ then, since $a_{n+1}>0,$ we have $$a_{n+1}>sqrt 3iff a_{n+1}^2>3iff$$ $$iff frac {3^2(1+a_n)^2}{(3+a_n)^2}>3iff$$ $$iff 3^2(1+a_n)^2 -3(3+a_n)^2>0iff$$ $$iff 6(a_n^2-3)>0.$$ So by induction we have $a_n>sqrt 3$ for all $nin Bbb N.$



                                    Since $3+a_n>a_n>0$ we have $$a_n>a_{n+1} iff (3+a_n)left(a_n-frac {3(1+a_n)}{3+a_n}right)>0 iff$$ $$iff 3a_n+a_n^2> 3+3a_niff a_n^2>3,$$ and we have already shown that $a_n^2>3.$ So $a_n>a_{n+1}$ for all $n.$



                                    Since $(a_n)_n$ is a decreasing sequence of positive terms it has a limit $Lgeq 0$ . And we have $$L=lim_{nto infty}a_{n+1}=lim_{nto infty}frac {3(1+a_n)}{3+a_n}= frac {3(1+L)}{3+L}.$$ So $L=frac {3(1+L)}{3+L}, $
                                    which implies $L^2=3.$ Now $(Lgeq 0land L^2=3)implies L=sqrt 3.$






                                    share|cite|improve this answer























                                      up vote
                                      0
                                      down vote










                                      up vote
                                      0
                                      down vote









                                      We have $a_1>0$ and from the definition of $a_{n+1}$ we have $a_n>0implies a_{n+1}>0 $ for any $n$. So by induction we have $a_n>0$ for all $nin Bbb N.$



                                      We have $a_1>sqrt 3$. If $a_n>sqrt 3$ then, since $a_{n+1}>0,$ we have $$a_{n+1}>sqrt 3iff a_{n+1}^2>3iff$$ $$iff frac {3^2(1+a_n)^2}{(3+a_n)^2}>3iff$$ $$iff 3^2(1+a_n)^2 -3(3+a_n)^2>0iff$$ $$iff 6(a_n^2-3)>0.$$ So by induction we have $a_n>sqrt 3$ for all $nin Bbb N.$



                                      Since $3+a_n>a_n>0$ we have $$a_n>a_{n+1} iff (3+a_n)left(a_n-frac {3(1+a_n)}{3+a_n}right)>0 iff$$ $$iff 3a_n+a_n^2> 3+3a_niff a_n^2>3,$$ and we have already shown that $a_n^2>3.$ So $a_n>a_{n+1}$ for all $n.$



                                      Since $(a_n)_n$ is a decreasing sequence of positive terms it has a limit $Lgeq 0$ . And we have $$L=lim_{nto infty}a_{n+1}=lim_{nto infty}frac {3(1+a_n)}{3+a_n}= frac {3(1+L)}{3+L}.$$ So $L=frac {3(1+L)}{3+L}, $
                                      which implies $L^2=3.$ Now $(Lgeq 0land L^2=3)implies L=sqrt 3.$






                                      share|cite|improve this answer












                                      We have $a_1>0$ and from the definition of $a_{n+1}$ we have $a_n>0implies a_{n+1}>0 $ for any $n$. So by induction we have $a_n>0$ for all $nin Bbb N.$



                                      We have $a_1>sqrt 3$. If $a_n>sqrt 3$ then, since $a_{n+1}>0,$ we have $$a_{n+1}>sqrt 3iff a_{n+1}^2>3iff$$ $$iff frac {3^2(1+a_n)^2}{(3+a_n)^2}>3iff$$ $$iff 3^2(1+a_n)^2 -3(3+a_n)^2>0iff$$ $$iff 6(a_n^2-3)>0.$$ So by induction we have $a_n>sqrt 3$ for all $nin Bbb N.$



                                      Since $3+a_n>a_n>0$ we have $$a_n>a_{n+1} iff (3+a_n)left(a_n-frac {3(1+a_n)}{3+a_n}right)>0 iff$$ $$iff 3a_n+a_n^2> 3+3a_niff a_n^2>3,$$ and we have already shown that $a_n^2>3.$ So $a_n>a_{n+1}$ for all $n.$



                                      Since $(a_n)_n$ is a decreasing sequence of positive terms it has a limit $Lgeq 0$ . And we have $$L=lim_{nto infty}a_{n+1}=lim_{nto infty}frac {3(1+a_n)}{3+a_n}= frac {3(1+L)}{3+L}.$$ So $L=frac {3(1+L)}{3+L}, $
                                      which implies $L^2=3.$ Now $(Lgeq 0land L^2=3)implies L=sqrt 3.$







                                      share|cite|improve this answer












                                      share|cite|improve this answer



                                      share|cite|improve this answer










                                      answered Nov 17 at 11:17









                                      DanielWainfleet

                                      33.4k31647




                                      33.4k31647






























                                           

                                          draft saved


                                          draft discarded



















































                                           


                                          draft saved


                                          draft discarded














                                          StackExchange.ready(
                                          function () {
                                          StackExchange.openid.initPostLogin('.new-post-login', 'https%3a%2f%2fmath.stackexchange.com%2fquestions%2f3000622%2fshow-that-the-sequence-a-1-3-and-a-n1-frac31a-n3a-n-converges-to%23new-answer', 'question_page');
                                          }
                                          );

                                          Post as a guest















                                          Required, but never shown





















































                                          Required, but never shown














                                          Required, but never shown












                                          Required, but never shown







                                          Required, but never shown

































                                          Required, but never shown














                                          Required, but never shown












                                          Required, but never shown







                                          Required, but never shown







                                          Popular posts from this blog

                                          Quarter-circle Tiles

                                          build a pushdown automaton that recognizes the reverse language of a given pushdown automaton?

                                          Mont Emei